use the subtitution rule

Jonkies

New member
Joined
Aug 24, 2020
Messages
3
Use the substitution rule to evaluate
∫\(\displaystyle ^∞_0\) ln?/(1+?^2) ?? .....................[edited]

∞ is the upper bound
0 is the lower bound

Can you please help me with that i couldn't find the correct answer
 
Last edited by a moderator:
Please use parentheses to clarify intent.

1/1+x^2 is NOT the same as 1/(1+x^2).

What substitution were you planning to use?

Why aren't you worried about convergence?
 
Well, you'll have to suggest what it is you did try and why you think it didn't work.

At x = 0, that argument in infinite. Will ANYTHING work? This is an important question.
At x = 1, that argument changes sign. Will that confuse anything?
 
don't ask integration, i cannot resist?

when i saw it the first time i knew he meant from [MATH]0[/MATH] to [MATH]\infty[/MATH]
 
Use the substitution rule to evaluate
∫\(\displaystyle ^∞_0\) ln?/(1+?^2) ?? .....................[edited]

∞ is the upper bound
0 is the lower bound

Can you please help me with that i couldn't find the correct answer
Is your problem:

\(\displaystyle \int^{\infty}_0 ln\left[\frac{x}{1+x^2}\right]dx\)

.................................or

\(\displaystyle \int^{\infty}_0 \frac{ln(x)}{1+x^2}dx\)

Please clarify.
 
This is what I got

u=1/x du=1/x^2


\(\displaystyle \int^{\infty}_0 \frac{ln(x)}{1+x^2}dx\)

ln(x)+C

=-1ln |u| |^{1}_0

= -1ln(1)-ln(0)+C
 
Last edited:
Let's see...

[math]u = \dfrac{1}{x}[/math]
[math]du = -\dfrac{1}{x^{2}}\;dx[/math] <== Your version has the wrong sign and is missing some notation.

[math]\int^{\infty}_{0}\dfrac{-ln(u)}{u^{2} + 1}\;du[/math] <== It is not clear at all how you managed simply 1/x in your integrand. 1) Shouldn't that be something in terms of "u"? 2) Where did the logarithm go? 3) How did you lose the "1" in the denominator. Please be MUCH more careful with your algebra. If you get through all the algebra correctly, lo, and behold, we didn't get anywhere at all. That substitution resulted in essentially the same problem. That's no good.

Technical Point: If you know u > 0, there is no need to write |u|.

You have glibly written "ln(0)". That's no good. This goes back to my original questions. Why aren't you worried about convergence? "ln(0)" doesn't mean anything.

In your last two statements, it appears that [math]\infty[/math] has turned into "1"? How did that happen under the transformation 1/x? And how did zero remain zero under that same transformation? Or, was it [math]\infty[/math] turned into 0 and 0 turned into 1? I can't tell.

Technical Point: This is a definite integral. What it "+C" doing there at the end?

Why are we constrained to Substitution? Frankly, "By Parts" looks more compelling, but we still have that Convergence problem.

Why don't we try a simpler problem and see if we are getting the concept of Substitution? Maybe [math]\int\dfrac{2x}{x^{2}+1}\;dx[/math]?
 
Top